LSAT and Law School Admissions Forum

Get expert LSAT preparation and law school admissions advice from PowerScore Test Preparation.

 Administrator
PowerScore Staff
  • PowerScore Staff
  • Posts: 8919
  • Joined: Feb 02, 2011
|
#23477
Complete Question Explanation

Weaken—CE,#%. The correct answer choice is (A)

In this stimulus, the author asserts a simple causal argument:
  • Premise: ..... In modern times, news is full of violence.

    Conclusion: ..... Violent crime is out of control.
This causal argument could be diagrammed as follows:
  • Cause ..... ..... :arrow: ..... Effect
    violent crime up ..... ..... news media now full of violent stories
Since the stimulus is followed by a weaken question stem, we know that the right answer is likely to provide an alternative cause, or a reason to doubt the causal link asserted by the author.

Answer choice (A): This is the correct answer choice, because it provides an alternative cause for all of the violent news stories: more comprehensive coverage. Maybe there isn't more violence today than there used to be—even if it's the same amount (or less), this answer choice provides a cause for the increased media coverage.

Answer choice (B): This answer choice plays no role in the author's conclusion, which only deals with violence "in our city."

Answer choice (C): The relative incidence of violence is not relevant to the author's asserted relationship between actual violence and media coverage of violence.

Answer choice (D): Like answer choice (D) above, this answer choice is incorrect because the proportionality presented is irrelevant to the question of whether violent crime in general has caused the extensive media coverage of violent crime. The portion of violent crime comprised by murders is not relevant to this inquiry.

Answer choice (E): The importance of the role of newspapers today versus the past neither strengthens nor weakens the author's assertion that the media's numerous stories on violent crime is attributable to a actual increase in violent crime.
 jrc3813
  • Posts: 53
  • Joined: Apr 16, 2017
|
#35852
When prephrasing I thought to look for an answer that would say something about newspapers providing more coverage which gives the impression that crime is up. B fits the prephrasing but can you help me eliminate E better? I eliminated it mostly because it says magazines instead of newspapers. But even if it said newspapers, would it still be wrong because even if they play a more important role today, it doesn't necessarily mean that they cover crime more? Basically we have to make an extra assumption that because they play a more important role, newspapers choose to provide more coverage overall? Maybe society looks to newspapers more today for crime information for whatever reason, but newspapers have not responded by increasing coverage. They're just going about business as usual. Would that be the reason to eliminate it?
 Adam Tyson
PowerScore Staff
  • PowerScore Staff
  • Posts: 5153
  • Joined: Apr 14, 2011
|
#36045
I think your first instinct here is the right one, jrc, and that's to eliminate answer E because it's talking about news magazines rather than newspapers. Back in 2000 when this test was administered, this would have been even more obvious than it is today, now that various media forms have blurred together so much and traditional print media have declined substantially. Back then, a newspaper and a magazine were clearly and obviously distinct from one another. That change over time may have contributed to your spending more time than you probably should have on this answer choice.

If E did say newspapers instead of news magazines, then your analysis would still be correct - playing a more important role is not the same as having more coverage. Maybe they play a more important role because other sources have diminished? Who knows? Since this answer does nothing to challenge the claim that crime has gotten out of control, it's a loser.

Good work!
 Leela
  • Posts: 63
  • Joined: Apr 13, 2019
|
#64268
Why is it that we're not looking to also weaken "to be safe from personal attack, one should not leave one's home except for absolute necessities?"
 Brook Miscoski
PowerScore Staff
  • PowerScore Staff
  • Posts: 418
  • Joined: Sep 13, 2018
|
#64270
Leela,

The main reason to focus on whether violent crime is out of control is that the stimulus only offers evidence to support that conclusion. (A) indicates why that evidence does not offer reliable support for the conclusion. The conclusion that we should cower in our homes is asserted on the claim that the first part of the conclusion (violence is out of control) is correct, so weakening the first part of the conclusion also weakens that subsequent claim. None of the remaining choices do a good job of attacking the claim that we should cower in our homes.
 Legallyconfused
  • Posts: 19
  • Joined: Oct 03, 2019
|
#71996
Hi!

I made a weird error in understanding what answer (A) was saying.
I took "comprehensive coverage" in (A) to mean that today's reporters are more detailed in reporting the crime stories, not that they are covering more crime stories. How should I have avoided this mistake?
Now that I reread (A) (after sadly getting this questions incorrect) I can see how "comprehensive coverage" means that they cover more stories. But in the moment I was like, "awesome, reporters today have stories that have more details about the events."
 Jeremy Press
PowerScore Staff
  • PowerScore Staff
  • Posts: 1000
  • Joined: Jun 12, 2017
|
#72103
Hi Legallyconfused,

So, a couple things: first, keep an eye out for terminological flexibility throughout the LSAT. There's not much you can do if you don't notice the flexibility inherent in a term the first time you read it. But just remembering that the LSAT is constantly testing whether you're aware of multiple meanings can lead you to look for flexible terms (especially when, in a situation like this, you might not see any answer choice that works!). If you learn something from a question about a term's flexibility, file it away for future usage. Will that term appear on every exam? No, but knowing it is better than not knowing it. And just knowing you've made that mistake before can prepare you to be vigilant about terms with multiple meanings.

Second, I think you can still get this question right even if you read answer choice A the way you have. The mistake in the stimulus is taking what the newspapers report as indicative of a change in the level of actual crime. If answer choice A just means the newspapers are covering more details of crimes than they did before, what does that tell us? That perhaps there aren't more crimes, there are just more details in the newspapers about them. That would also weaken the conclusion! So I actually think either way you read answer choice A, you can still arrive at the right place.

I hope this helps!

Jeremy

Get the most out of your LSAT Prep Plus subscription.

Analyze and track your performance with our Testing and Analytics Package.